Satz von Goldstone, Symmetriebrechung und das Heisenberg-Modell

Ich erforsche derzeit Symmetriebrechung und Goldstones Theorem für ein Projekt in meinem dritten Jahr meines Studiums der Theoretischen Physik. Mein Wissen stammt also nicht aus einer formellen Lehre, sondern aus meiner eigenen Forschung.

Ich begann mit dem Versuch, das Theorem von Goldstone zu verstehen, und nach dem, was ich verstehe, ist es die Idee, dass Sie masselose Skalarfelder (Goldstone-Bosonen) erhalten, wenn eine kontinuierliche Symmetrie gebrochen wird. Ich habe die Mathematik davon durchgegangen und es scheint Sinn zu machen.

Ich schaue mir jedoch das Heisenberg-Modell als eine Art reales Beispiel für Goldstones Theorem an und stoße auf Probleme. Ich denke, ich habe keine Frage an sich, sondern eher, um zu sehen, ob mein Verständnis richtig ist. Das Heisenberg-Modell besagt also, dass der Hamilton-Operator aus den Spins der nächsten Nachbarn in einem Gitter besteht. Offensichtlich ist dieser Hamiltonian unter Rotation symmetrisch (wenn Sie alle Spins um Theta drehen, bleibt die Nettoenergie gleich?). Der Grundzustand wäre der Zustand, in dem alle Spins in die gleiche Richtung zeigen, und es gibt natürlich unendlich viele davon, da sie in jede Richtung zeigen können, vorausgesetzt, sie zeigen alle in die gleiche Richtung. Ich habe dann gehört, dass die "Auswahl" eines Grundzustands diese Symmetrie spontan bricht, weil Sie ' Sind Sie jetzt von einer unendlichen Anzahl möglicher Grundzustände, die bei Rotation unveränderlich sind, in einen einzigen Zustand zusammengebrochen, und wenn Sie daher alle Spins drehen, wäre es nicht dieser spezifische Zustand, den Sie ausgewählt haben? Außerdem, wo kommt der Satz von Goldstone ins Spiel? Ich habe etwas über Spinwellen gehört, sind das in diesem Fall die Goldstone-Bosonen?

Ich hoffe, jemand kann mir helfen, meine Fragen zu beantworten oder mich in die richtige Richtung weisen. Ich habe versucht, mich klar zu erklären, ob das erreicht wurde oder nicht, ist eine andere Geschichte.

Antworten (2)

Das Heisenberg-Modell ist eigentlich ein Beispiel für eine Ausnahme vom Standard-Goldstone-Theorem für die relativistische QFT. Im Standardfall erwarten wir, dass jede gebrochene Symmetrie eine lückenlose Mode mit linearer Dispersion bei kleinen Impulsen liefert. Dies gilt im Allgemeinen nicht für nichtrelativistische Systeme wie das Heisenberg-Modell. Betrachten Sie den Hamilton-Operator

H = J N = 1 N S N S N + 1 ,
wobei wir periodische Randbedingungen annehmen S N + 1 = S 1 . Hier beschreiben wir der Einfachheit halber eher eine Spinkette als ein Gitter. Da der Hamilton-Operator die nächsten Nachbarn durch ein Skalarprodukt koppelt, weist der Hamilton-Operator selbst eine Rotationssymmetrie um die auf X , j , Und z Achsen. Im Grundzustand dieses Systems zeigen alle Spins in die gleiche Richtung, die wir als die wählen werden + z Richtung; das ist,
| 0 = | , . . . , .
Somit hat das Vakuum oder der Grundzustand der Theorie eine Vorzugsrichtung im Raum gewählt und die ursprüngliche Rotationssymmetrie gebrochen. Eher, | 0 ist nur bei Drehungen um die invariant z -Achse. Sie können diese Tatsache selbst überprüfen, indem Sie mit den Standard-SU(2)-Rotationsoperatoren (einer für jeden Gitterplatz) auf den Grundzustand einwirken. Wir sagen, dass die ursprüngliche SO(3)-Symmetrie zu SO(2) gebrochen wurde. Wir haben in diesem Fall zwei gebrochene Symmetrien, die Drehungen um die entsprechen X Und j Achsen.

Betrachten Sie nun den folgenden Zustand

| k = N = 1 N e ich k N | . . . , , N , , . . . ,
das ist eine lineare Kombination von Zuständen mit der N -ten Spin nach unten geklappt. Man kann zeigen, dass dieser Zustand ein Energieeigenzustand mit Anregungsenergie (über dem Grundzustand) ist
Δ E = 2 J ( 1 cos ( k ) ) ,
die lückenlos und quadratisch ist k für klein k . Diese Anregung wird als Magnon bezeichnet. Das Merkwürdige hier ist, dass wir zwei Symmetrien gebrochen haben, aber nur eine einzige Anregung mit quadratischer statt linearer Dispersion gefunden haben. Der Grund dafür ist, dass die beiden Symmetrien, die wir gebrochen haben, nicht unabhängig sind, da die Erzeuger dieser Symmetrien die bekannten Kommutierungsbeziehungen erfüllen
[ σ ich , σ J ] = 2 ich ϵ ich J k σ k ,
damit insbesondere
[ σ X , σ j ] = 2 ich σ z .
Durch die Durchführung von Drehungen um die X Und j Achsen können wir eine Drehung um die erzeugen z -Achse.

Die Anzahl der kaputten Generatoren ist genau gleich der Anzahl der Goldstone-Bosonen if

0 | [ Q ich , Q J ] 0 = 0 ,
für alle gebrochenen Symmetriegeneratoren Q . Jetzt fragen Sie sich vielleicht, was an der Lorentz-Invarianz so besonders ist. Erinnern Sie sich, dass der Satz von Noether uns sagt, dass erhaltene Ladungen (dh Symmetriegeneratoren) als Integral eines erhaltenen Stroms geschrieben werden können
Q = J 0 ( X ) D 3 X ,
wohingegen die Lorentz-Invarianz verlangt, dass die Erwartungswerte von Objekten mit freien Lorentz-Indizes verschwinden müssen
J 0 ( X ) = 0 ,
Dadurch wird sichergestellt, dass die Anzahl der Goldstone-Bosonen immer gleich der Anzahl der Generatoren für gebrochene Symmetrie in einer relativistischen Theorie ist.

Möglicherweise finden Sie die folgende Referenz nützlich (die Quelle der meisten dieser Informationen). Spontane Symmetriebrechung

@ Evan Rule, wenn k = 0, die Energie des Zustands | k = 0 ist dasselbe wie der geordnete Grundzustand, also ist der "Grundzustand" nicht immer geordnet, er "oszilliert", der Zustand mit umgedrehtem Spin ist auch der Grundzustand, richtig?

Ja, du hast recht. Wenn Sie in diesem Fall einen bestimmten Grundzustand wählen, wählen Sie eine bestimmte Richtung, in die alle Spins zeigen sollen. Anregungen weg von diesem Grundzustand, dh Wellen, bei denen die Spins der Teilchen vom gewählten Grund weg schwingen Staatsrichtung, sind die Goldstone-Bosonen.